LSAT and Law School Admissions Forum

Get expert LSAT preparation and law school admissions advice from PowerScore Test Preparation.

 PeterC123
  • Posts: 26
  • Joined: Dec 27, 2016
|
#31628
Hi,

I thought the passage was saying that there is a causal relation between tv and growth in homicide rates. I choose A b/c I thought that A showed without the cause there is no effect. Is it wrong b/c the passage never mentioned violent tv, just tv in general?

I did not pick B b/c I thought it was too good to be true since it just paraphrased the argument...

Am I understanding my fault here or am I looking at this from a wrong angle?

Thanks,
 Kristina Moen
PowerScore Staff
  • PowerScore Staff
  • Posts: 230
  • Joined: Nov 17, 2016
|
#31648
Peter,

Answer Choice (B) does not paraphrase the argument. Be careful. Sometimes (often!) we fill in gaps in arguments ourselves. That's why Assumption questions can be so difficult - we often fill in the assumptions as we read an argument. The columnist's conclusion is that the advent of television (i.e. the beginning of television) helps explain why the growth in homicide rates began earlier in urban areas than rural ones. Since this is a Strengthen question, any answer choice that shows how/why TV causes homicide will strengthen the conclusion. Answer Choice (A) is incorrect because TV programs could simply reflect the community (e.g. perhaps people watch "family" shows like Sesame Street in already low-crime areas - this does not help show that TV causes homicide).
 martinbeslu
  • Posts: 49
  • Joined: Aug 09, 2017
|
#44910
I also didn't understand why A is correct. The correlation in the stimulus is about television sets is between television sets becoming popular in homes and homicide rates. There is no discussion about how much, if any violence there was on TV. If answer choice B said that television sets becoming popular in homes is a cause, not an effect, of the violence in society then this would seem to support the argument.

Answer choice E says that increasing one's amount of leisure time increases one's inclination to act violently. I would think that watching tv is considered leisure rather than research or work. As far as assumptions go, I would think that is very reasonable, especially compared to the assumption in A that there was violence on TV. Can you help me understand what makes the assumptions in A allowed but not the assumptions in E? As always, thank you for helping me to understand these concepts!!!
 Adam Tyson
PowerScore Staff
  • PowerScore Staff
  • Posts: 5374
  • Joined: Apr 14, 2011
|
#44953
To be clear, martinbeslu, the correct answer here is B, not A. I wasn't sure from your post if you had that or if there was some confusion.

There are a few things going on in your analysis that deserve some attention, and I think the first is to look at the difference between a Strengthen question, like this one ("most supports" the argument) and a Justify the Conclusion question (where the answer proves the conclusion). Answer B does not justify the conclusion, but it helps, because it links TV causally to violence in society. Sure, there may not have been much violence on TV back when folks started getting TVs, but at least answer B is telling us that something on TV does cause, and is not caused by, societal violence. It doesn't matter that it falls far short of proving the conclusion, because this is only a Strengthen question. All we have to do is help. Answer B helps, without making any additional assumptions or bringing in any outside info.

On the flip side, you're giving way too much help to answer E! Perhaps you are right that watching TV qualifies as leisure time, but where do we get the information that watching TV increases your leisure time? Maybe we just swap out the time we would have been doing something else for TV time? That's not an assumption - I'm not saying that DID happen, or that it must be the case - but a consideration that shows that answer E requires additional information that was not given to us. We don't know anything about an increase in leisure time, only about TVs showing up in homes. The correct answer will help the argument all by itself, with no additional info, and answer E needs more before it gets there.

Answer A is wrong for a couple of reasons. First, the number of programs doesn't seem relevant, because it could be more about the simple presence of TV, or perhaps the level of violence in a given program. More importantly, in my opinion, is that this answer only tells us something about a further correlation, without telling us whether it's causal. That might help some - where the cause is less, the effect is less, perhaps - but since answer B is directly causal it is much stronger than whatever answer A's mere correlation can do for us. Remember to pick the one that does the most to support the claim, not just one that might support it some! A further correlation is never going to be as powerful as a direct claim of causality.

Take another look and let us know if that makes it clearer! Keep up the good work!
 martinbeslu
  • Posts: 49
  • Joined: Aug 09, 2017
|
#45147
Hi Adam. Thank you for the help! It totally makes sense that a causal relationship is stronger support than a correlation. I am completely with you in understanding that all answer choices other than B and E can be quickly eliminated. The part I am struggling with is figuring out which assumptions we are allowed to make. Answer choice B has to assume that there was violence on television (which is never discussed in the stimulus) or otherwise it does nothing for the argument. Answer choice E has to assume that when “television sets became popular”, “leisure time increased”, which seems to be at least somewhat supported by the word “popular”. Can you please help me understand what makes violence on tv a more reasonable assumption than television sets becoming popular increasing one’s amount of leisure time? Figuring out what assumption the test makers want us to make is what I struggle with more than anything on difficult questions.
 Daniel Stern
PowerScore Staff
  • PowerScore Staff
  • Posts: 80
  • Joined: Feb 07, 2018
|
#45158
Martin:

It's true, the portrayal of violence on TV is not mentioned in the stimulus. But recall that this is a Strengthen Question, from our Help family. So the logic flows from the Answer Choices, which we hold as true -- the Stem tells us to do so (i.e. "Which one of the following, if true...") up to the stimulus. It's ok that B includes portrayal of violence on TV -- if that is true, meaning there is such a thing, and that portrayal bears a causal relationship to violence in society, that helps the author's argument that TVs showing up caused more murders.

I really like Adam's explanation for Answer E above:
On the flip side, you're giving way too much help to answer E! Perhaps you are right that watching TV qualifies as leisure time, but where do we get the information that watching TV increases your leisure time? Maybe we just swap out the time we would have been doing something else for TV time? That's not an assumption - I'm not saying that DID happen, or that it must be the case - but a consideration that shows that answer E requires additional information that was not given to us. We don't know anything about an increase in leisure time, only about TVs showing up in homes. The correct answer will help the argument all by itself, with no additional info, and answer E needs more before it gets there.
So for E to strengthen the argument, we'd need to hold E true AND add other information. That's the sort of assumption the test-makers don't want you to make. Hold B as true: author's argument is strengthened. Hold E as true: still need more information to help author's argument.

Hope that is helpful.
Dan
 gen2871
  • Posts: 47
  • Joined: Jul 01, 2018
|
#47763
Hi, dear LSAT masters:

I don't quite get it why D is not a good choice. the earlier one is exposed to violence on television, is this referring age? otherwise i see this a perfect fit. I get B is the correct answer, reemphasize the cause is the cause strengthens the argument itself. but I don't quite get why D is incorrect. Please specify. thank you.
 Adam Tyson
PowerScore Staff
  • PowerScore Staff
  • Posts: 5374
  • Joined: Apr 14, 2011
|
#47797
Answer D's reference to "earlier" is indeed a reference to age, gen2871. It means the same as "the younger a person is when they are first exposed to violence on television...", and of course the age of any given individual is not the issue here. It's not about the date that a group is exposed, but the age at which an individual is exposed.

Looks to me like you understand why that makes it a wrong answer! Well done.
 dbrowning
  • Posts: 26
  • Joined: Jun 18, 2019
|
#67561
Does the language "help explain" necessitate a causal relationship here? I assume that, if X is only correlated with Y, X cannot "help explain" Y. The most X could do, with this relationship, is to help indicate that Y is also occurring. Is that accurate? I could see this kind of language coming up again on future questions.
 Adam Tyson
PowerScore Staff
  • PowerScore Staff
  • Posts: 5374
  • Joined: Apr 14, 2011
|
#68464
You got it, dbrowning! "Helps explain" is indicating an implied causal relationship. If the presence of A helps to explain the presence of B, as weak as that claim is, it does imply that A is at least partially to blame for B - it's a causal factor, if not the sole cause. To strengthen that, anything that adds to that alleged causal relationship would be good. Answer B eliminates a possible reversed cause and effect relationship between societal violence and at least some kinds of TV programs, and that elimination makes the argument just a little bit better.

Nice work!

Get the most out of your LSAT Prep Plus subscription.

Analyze and track your performance with our Testing and Analytics Package.